This discussion board is a place to discuss our Daily Challenges and the math and science
related to those challenges. Explanations are more than just a solution — they should
explain the steps and thinking strategies that you used to obtain the solution. Comments
should further the discussion of math and science.
When posting on Brilliant:
Use the emojis to react to an explanation, whether you're congratulating a job well done , or just really confused .
Ask specific questions about the challenge or the steps in somebody's explanation. Well-posed questions can add a lot to the discussion, but posting "I don't understand!" doesn't help anyone.
Try to contribute something new to the discussion, whether it is an extension, generalization or other idea related to the challenge.
Stay on topic — we're all here to learn more about math and science, not to hear about your favorite get-rich-quick scheme or current world events.
Markdown
Appears as
*italics* or _italics_
italics
**bold** or __bold__
bold
- bulleted - list
bulleted
list
1. numbered 2. list
numbered
list
Note: you must add a full line of space before and after lists for them to show up correctly
try use trigonometric substitution, Let a=tan2A,b=tan2B,c=tan2C,d=tan2D
and the equation becomes cos2A+cos2B+cos2C+cos2D=3
Observe that cos2A=sin2B+sin2C+sin2D, then use AM-GM, we will get cos2A≥3.(sinB.sinC.sinD)32
Use the similar way to get cos2B,cos2C,cos2D
Last, multiplying 4 inequalities that you have got, and you can find easily that abcd≤811
This ain't nice but does the trick.
Subject to that condition, there exist a maximum of abcd (We can do an argument that it is a function from a compact space to another compact space, so it must have a maximum.).
Suppose we have that maximum.Let x=1+a1 and similarly y,z and w for b,c,d. Then a=x1−x and a1=1−xx. We have x+y+z+w=3 and 0≤x,y,z,w≤1 so x+y+2≥x+y+z+w=3 so 2≥x+y≥1 so x+y=1+e with e in the interval [0,1]. (a variable, not the constant)
Now substitute y=1+e−x and a1=1−xx and similarly for b in ab1.Then
This function achieves maximum when 1−x=x−e, that is x=1+e−x=y, that is a=b. So, if we have (a,b,c,d) that satisfy the hypothesis and is a maximum of abcd then it must have a=b,else setting x′=y′=(x+y)/2 would make another (a′,b′,c,d) that satisfy the hypothesis with a strictly greater product. So, a=b. In the same manner, b=c=d. So, by hypothesis, a=31. Thus the maximum of abcd is (31)4=811
One more solution , which is relied on again AM-GM inequality .
Substitute ,
1+a1=x,1+b1=y,1+cc=z,1+dd=w.
where x+y+z+w=3.
We get a=x1−x,b=y1−y,c=z1−z,d=w1−w
Now ,
We want to prove that abcd≤811
So it suffice to prove that ,
81(1−x)(1−y)(1−z)(1−w)≤xyzw
since 3=x+y+z+w
thus following inequality is equivalent to
(y+z+w−2x)(x+z+w−2y)(x+y+w−2z)(x+y+z−2w)≤xyzw
Now we again do a obvious substitution , that is
m=y+z+w−2x,n=x+z+w−2y,o=x+y+w−2z,p=x+y+z−2w
So , again the inequality is equivalent to ,
mnop≤3m+n+o⋅3n+o+p⋅3m+n+p⋅3m+o+p
which is obvious by AM-GM :)
HINT: One easy (but not so clever) way that works is to use Jensen's Inequality, and then use AM-GM. Although there are probably better ways to do this.
Thanks solved it.. But can you please tell me what are weights??..I studied wikipedia article on jensens inequality and it states something about weight associated with each number..I used the formula assuming equal weights and got the answer..
And we had to use the fact that y=x+11 is a convex function??
Kindly can u please explain it to me in detail..
I think this comment gets at the intuition that the symmetric case of a=b=c=d will be critical. This is probably how the question was written in the first place, but it does not actually answer the question or provide a proof. Some sort of convexity inequality is clearly called for, and playing around with Jensen's and/or power means will eventually lead to the lovely arguments above.
I agree, and in fact, Alvin's comment can be turned into a proof that one can easily do in one's own head. So all the downvotes were perhaps a bit unnecessary. The argument I'm going to sketch is actually a variant of what Diego proposed.
The question can be rephrased as follows: maximize the function F(x,y,z,w)=f(x)+f(y)+f(z)+f(w), where f(t)=log(t1−1), subject to the constraints x+y+z+w=3 and 0<x,y,z,w<1. One does need a small argument to show that Fdoes achieve a maximum somewhere on this set, but once we know this, it follows by standard multi-variable calculus that at the point where the maximum is achieved, the gradient of F must be proportional to the vector (1,1,1,1) (which is the gradient of the function H(x,y,z,w)=x+y+z+w−3 that defines our hyperplane). This implies that x=y=z=w at the point where the maximum is achieved, and then Alvin's computation finishes the proof.
@John Smith
–
I like the idea of inverting the 1+t1 to t1−1 and then using a sum of logs rather than the original product. If I understand correctly, you don't even need to compute the derivatives since x,y,z,w will all be the same solution of the same equation, f′(t)=k, where k is the proportionality factor, and then we can solve for a,b,c,d at the maximum using the original linear constraint and a=b=c=d.
The problem at first seems like it hinges on convexity, but the Lagrange multiplier method doesn't seem to involve it. Is convexity just a nice Olympiad-style shortcut or is it still operating in this solution, too?
@Eric Edwards
–
Well, I was actually a bit hasty in my claim. It is true that x,y,z,w will all be solutions of the same equation, and you are right that to show this one doesn't even need to compute any derivatives. Unfortunately, this doesn't immediately guarantee that they have to be the same solution of this equation.
However, if you look at both the first and the second derivatives, you will see that in fact, x,y,z,w have to be the same because the equation f′(t)=k can have at most two solutions, and f′′(t) is nonpositive for only one of those two solutions. (The point is that the second derivative can't be positive if you are at a point where a local max is achieved.)
As far as convexity goes, it is certainly not just an Olympiad-style shortcut, but the problem is that I don't see a way to immediately apply convexity here (and this is why all the other solutions that were posted required some additional calculations). For instance, if the function f(t)=log(1/t−1) happened to be concave, we would be done immediately, but it just isn't: its second derivative has one sign on (0,1/2) and the opposite sign on (1/2,1).
Easy Math Editor
This discussion board is a place to discuss our Daily Challenges and the math and science related to those challenges. Explanations are more than just a solution — they should explain the steps and thinking strategies that you used to obtain the solution. Comments should further the discussion of math and science.
When posting on Brilliant:
*italics*
or_italics_
**bold**
or__bold__
paragraph 1
paragraph 2
[example link](https://brilliant.org)
> This is a quote
\(
...\)
or\[
...\]
to ensure proper formatting.2 \times 3
2^{34}
a_{i-1}
\frac{2}{3}
\sqrt{2}
\sum_{i=1}^3
\sin \theta
\boxed{123}
Comments
You can do it like this too :
1+a1+1+b1+1+c1+1+d1=3 1+a1=1+bb+1+cc+1+dd Now using AM - GM 1+a1=1+bb+1+cc+1+dd≥33(1+b)(1+c)(1+d)bcd
Thus ,
1+a1≥33(1+b)(1+c)(1+d)bcd Similarly we get , 1+b1≥33(1+a)(1+c)(1+d)acd 1+c1≥33(1+b)(1+a)(1+d)bcd 1+d1≥33(1+b)(1+c)(1+a)bcd
Multiplying them all together , gives 1≥81abcd As desired . :)
I have one more similar type of problem . You may like to try it .
Let x1,x2,⋯,xn be positive real numbers satisfying i=1∑i=nxi+19981=19981 Prove that , n−1(x1x2x3.....xn)n1≥1998
Log in to reply
Thanks..!
Log in to reply
good solution... similar with my solution, but without trigonometric substitution (y)
try use trigonometric substitution, Let a= tan2A,b= tan2B,c= tan2C,d= tan2D and the equation becomes cos2A+cos2B+cos2C+cos2D=3 Observe that cos2A=sin2B+sin2C+sin2D, then use AM-GM, we will get cos2A≥3.(sinB.sinC.sinD)32 Use the similar way to get cos2B,cos2C,cos2D Last, multiplying 4 inequalities that you have got, and you can find easily that abcd≤811
This ain't nice but does the trick. Subject to that condition, there exist a maximum of abcd (We can do an argument that it is a function from a compact space to another compact space, so it must have a maximum.).
Suppose we have that maximum.Let x=1+a1 and similarly y,z and w for b,c,d. Then a=x1−x and a1=1−xx. We have x+y+z+w=3 and 0≤x,y,z,w≤1 so x+y+2≥x+y+z+w=3 so 2≥x+y≥1 so x+y=1+e with e in the interval [0,1]. (a variable, not the constant)
Now substitute y=1+e−x and a1=1−xx and similarly for b in ab1.Then
ab1=(1−x)(1−y)xy=(1−x)(x−e)x(1+e−x)=x−x2+(x−1)ex−x2+xe=x−x2+(x−1)ee=1+(1−x)(x−e)e
This function achieves maximum when 1−x=x−e, that is x=1+e−x=y, that is a=b. So, if we have (a,b,c,d) that satisfy the hypothesis and is a maximum of abcd then it must have a=b,else setting x′=y′=(x+y)/2 would make another (a′,b′,c,d) that satisfy the hypothesis with a strictly greater product. So, a=b. In the same manner, b=c=d. So, by hypothesis, a=31. Thus the maximum of abcd is (31)4=811
One more solution , which is relied on again AM-GM inequality .
Substitute , 1+a1=x,1+b1=y,1+cc=z,1+dd=w. where x+y+z+w=3. We get a=x1−x,b=y1−y,c=z1−z,d=w1−w Now , We want to prove that abcd≤811 So it suffice to prove that , 81(1−x)(1−y)(1−z)(1−w)≤xyzw since 3=x+y+z+w thus following inequality is equivalent to (y+z+w−2x)(x+z+w−2y)(x+y+w−2z)(x+y+z−2w)≤xyzw Now we again do a obvious substitution , that is m=y+z+w−2x,n=x+z+w−2y,o=x+y+w−2z,p=x+y+z−2w So , again the inequality is equivalent to , mnop≤3m+n+o⋅3n+o+p⋅3m+n+p⋅3m+o+p which is obvious by AM-GM :)
HINT: One easy (but not so clever) way that works is to use Jensen's Inequality, and then use AM-GM. Although there are probably better ways to do this.
Log in to reply
Thanks solved it.. But can you please tell me what are weights??..I studied wikipedia article on jensens inequality and it states something about weight associated with each number..I used the formula assuming equal weights and got the answer.. And we had to use the fact that y=x+11 is a convex function?? Kindly can u please explain it to me in detail..
Applying AM-HM inequality 4a+1+b+1+c+1+d+1≥a+11+b+11+c+11+d+114. ⇒a+b+c+d≥34. Now applying AM-GM, 4a+b+c+d≥(abcd)41 ⇒811≥abcd.
I think I am wrong in the last 2 steps.Please correct me...
Log in to reply
No. If a+b+c+d is greater than 34 and 4(abcd)41, you cannot decide anything about the relation between the latter two.
Log in to reply
That's what I was wondering!
either try Jensen's inequality or try to take a>b>c>d, and try to bound the value of a.
\frac{1}{1+a}+\frac{1}{1+b}+\frac{1}{1+c}+\frac{1}{1+d}=3⇒\frac{1}{1+a}=(1-\frac{1}{1+b})+(1-\frac{1}{1+c})+(1-\frac{1}{1+d})=\frac{b}{1+b}+\frac{c}{1+c}+\frac{d}{1+d}.Applying AM-HM inequality:\frac{b}{1+b}+\frac{c}{1+c}+\frac{d}{1+d}\frac{1}{1+a}≥3\sqrt[3]{\frac{bcd}{(b+1)(c+1)(d+1)}}⇒\frac{1}{1+a}.\frac{1}{1+b}.\frac{1}{1+c}.\frac{1}{1+d}≥\frac{81abcd}{(a+1)(b+1)(c+1)(d+1}⇒ abcd≤\frac{1}{81}
Log in to reply
1+a1+1+b1+1+c1+1+d1=3⇒1+a1=(1−1+b1)+(1−1+c1)+(1−1+d1)=1+bb+1+cc+1+dd
Applying AM-HM inequality:
1+bb+1+cc+1+dd1+a1≥33(b+1)(c+1)(d+1)bcd⇒1+a1.1+b1.1+c1.1+d1≥(a+1)(b+1)(c+1)(d+1)81abcd⇒abcd≤811
3=43+43+43+43
43=1+a1=1+b1=1+c1=1+d1
341=1+a1=1+b1=1+c1=1+d1
34=1+a=1+b=1+c=1+d
So, a=b=c=d=31
Then, abcd=(31)4
abcd=811
Log in to reply
I think this comment gets at the intuition that the symmetric case of a=b=c=d will be critical. This is probably how the question was written in the first place, but it does not actually answer the question or provide a proof. Some sort of convexity inequality is clearly called for, and playing around with Jensen's and/or power means will eventually lead to the lovely arguments above.
Log in to reply
I agree, and in fact, Alvin's comment can be turned into a proof that one can easily do in one's own head. So all the downvotes were perhaps a bit unnecessary. The argument I'm going to sketch is actually a variant of what Diego proposed.
The question can be rephrased as follows: maximize the function F(x,y,z,w)=f(x)+f(y)+f(z)+f(w), where f(t)=log(t1−1), subject to the constraints x+y+z+w=3 and 0<x,y,z,w<1. One does need a small argument to show that F does achieve a maximum somewhere on this set, but once we know this, it follows by standard multi-variable calculus that at the point where the maximum is achieved, the gradient of F must be proportional to the vector (1,1,1,1) (which is the gradient of the function H(x,y,z,w)=x+y+z+w−3 that defines our hyperplane). This implies that x=y=z=w at the point where the maximum is achieved, and then Alvin's computation finishes the proof.
Log in to reply
1+t1 to t1−1 and then using a sum of logs rather than the original product. If I understand correctly, you don't even need to compute the derivatives since x,y,z,w will all be the same solution of the same equation, f′(t)=k, where k is the proportionality factor, and then we can solve for a,b,c,d at the maximum using the original linear constraint and a=b=c=d.
I like the idea of inverting theThe problem at first seems like it hinges on convexity, but the Lagrange multiplier method doesn't seem to involve it. Is convexity just a nice Olympiad-style shortcut or is it still operating in this solution, too?
Log in to reply
x,y,z,w will all be solutions of the same equation, and you are right that to show this one doesn't even need to compute any derivatives. Unfortunately, this doesn't immediately guarantee that they have to be the same solution of this equation.
Well, I was actually a bit hasty in my claim. It is true thatHowever, if you look at both the first and the second derivatives, you will see that in fact, x,y,z,w have to be the same because the equation f′(t)=k can have at most two solutions, and f′′(t) is nonpositive for only one of those two solutions. (The point is that the second derivative can't be positive if you are at a point where a local max is achieved.)
As far as convexity goes, it is certainly not just an Olympiad-style shortcut, but the problem is that I don't see a way to immediately apply convexity here (and this is why all the other solutions that were posted required some additional calculations). For instance, if the function f(t)=log(1/t−1) happened to be concave, we would be done immediately, but it just isn't: its second derivative has one sign on (0,1/2) and the opposite sign on (1/2,1).